Calculus of Variations; Maximum enclosed area problem.

In summary: \, dx \le c\\\\& & \end{array}$$where the indices ##x_1, y_1## are the ends of the string and ##x_2, y_2## are the coordinates of the midpoint of the string between the ends ##x_1, y_1##.
  • #1
Aaron Curran
33
0
The problem reads: "You are given a string of fixed length l with one end fastened at the origin O, and you are to place the string in the (x, y) plane with its other end on the x-axis in such a way as to maximise the area between the string and the x axis. Show that the required shape is a semicircle."
Hint: Show that the area can be written as
$$J(y) = \frac{1}{c}\int_{0}^{l} y(s)\sqrt{1 + (y'(s))^2}ds$$
Where s is the arc-length along the string, and then use the following first integral of the Euler-Lagrange equation
$$F - y'\frac{\partial F}{\partial y'} = C$$
Where ##y' = dy/ds## and ##F=y\sqrt{1 + (y')^2}##

My attempt at a solution:
I know that the rules of this forum state that saying I really don't have a clue what to do is not allowed but I genuinely have been stuck on this for all of today and for hours last night as well. I know that if y were a function of x, then the following would work
$$\frac{\triangle{s^2}}{\triangle{x^2}} = 1 + \frac{\triangle{y^2}}{\triangle{x^2}}$$
and then taking the limit as x tends to 0 would give
$$ds = \sqrt{1 + (y')^2}dx$$

But due to the question stating that y is a function of s, the arc-length. I have absolutely no clue where to go, I simply can not see how it is shown. Any help would be much appreciated, thank you!
 
Physics news on Phys.org
  • #2
Aaron Curran said:
The problem reads: "You are given a string of fixed length l with one end fastened at the origin O, and you are to place the string in the (x, y) plane with its other end on the x-axis in such a way as to maximise the area between the string and the x axis. Show that the required shape is a semicircle."
Hint: Show that the area can be written as
$$J(y) = \frac{1}{c}\int_{0}^{l} y(s)\sqrt{1 + (y'(s))^2}ds$$
Where s is the arc-length along the string, and then use the following first integral of the Euler-Lagrange equation
$$F - y'\frac{\partial F}{\partial y'} = C$$
Where ##y' = dy/ds## and ##F=y\sqrt{1 + (y')^2}##

My attempt at a solution:
I know that the rules of this forum state that saying I really don't have a clue what to do is not allowed but I genuinely have been stuck on this for all of today and for hours last night as well. I know that if y were a function of x, then the following would work
$$\frac{\triangle{s^2}}{\triangle{x^2}} = 1 + \frac{\triangle{y^2}}{\triangle{x^2}}$$
and then taking the limit as x tends to 0 would give
$$ds = \sqrt{1 + (y')^2}dx$$

But due to the question stating that y is a function of s, the arc-length. I have absolutely no clue where to go, I simply can not see how it is shown. Any help would be much appreciated, thank you!

Your expression for [itex]ds[/itex] is confusing you. The relationship between [itex]ds[/itex] and [itex]dx[/itex] is:

[itex]ds = \sqrt{1 + (\frac{dy}{dx})^2} dx[/itex]

What you wrote is incorrect, because [itex]y' \equiv \frac{dy}{ds}[/itex], not [itex]\frac{dy}{dx}[/itex]. And what you want is [itex]dx[/itex] in terms of [itex]ds[/itex], not [itex]ds[/itex] in terms of [itex]dx[/itex]. So how do you find [itex]dx[/itex] in terms of [itex]ds[/itex] and [itex]dy[/itex]?

Now, you plug in your answer for [itex]dx[/itex] in the expression for the area under the curve, which is just [itex]\int y dx[/itex].
 
Last edited:
  • #3
Aaron Curran said:
The problem reads: "You are given a string of fixed length l with one end fastened at the origin O, and you are to place the string in the (x, y) plane with its other end on the x-axis in such a way as to maximise the area between the string and the x axis. Show that the required shape is a semicircle."
Hint: Show that the area can be written as
$$J(y) = \frac{1}{c}\int_{0}^{l} y(s)\sqrt{1 + (y'(s))^2}ds$$
Where s is the arc-length along the string, and then use the following first integral of the Euler-Lagrange equation
$$F - y'\frac{\partial F}{\partial y'} = C$$
Where ##y' = dy/ds## and ##F=y\sqrt{1 + (y')^2}##

My attempt at a solution:
I know that the rules of this forum state that saying I really don't have a clue what to do is not allowed but I genuinely have been stuck on this for all of today and for hours last night as well. I know that if y were a function of x, then the following would work
$$\frac{\triangle{s^2}}{\triangle{x^2}} = 1 + \frac{\triangle{y^2}}{\triangle{x^2}}$$
and then taking the limit as x tends to 0 would give
$$ds = \sqrt{1 + (y')^2}dx$$

But due to the question stating that y is a function of s, the arc-length. I have absolutely no clue where to go, I simply can not see how it is shown. Any help would be much appreciated, thank you!

I don't understand the formulation. If I were doing it I would regard it as a maximization problem with a constraint. For a string of length ##c## we have the problem
$$\begin{array}{rl}
\max &J = \int_0^{x_1} y \, dx \\
\text{subject to} & \int_0^{x_1} \sqrt{1 + y'^2} \, dx = c\\
\text{and}& y(0) = y(x_1) = 0
\end{array}
$$
We could convert this to an unconstrained problem using a Lagrange multiplier method;
$$\max K = \int_0^{x_1} L(y,y') \, dx \equiv \int_0^{x_1} y \, dx + u \int_0^{x_1} \sqrt{1+y'^2} \, dx $$
with boundary conditions ##y(0) = y(x_1) = 0##.
Here, the real constant ##u## is the Lagrange multiplier and ##L(y,y') = y + u \sqrt{1 + y'^2}.##
 
  • #4
Ray Vickson said:
I don't understand the formulation. If I were doing it I would regard it as a maximization problem with a constraint. For a string of length ##c## we have the problem
$$\begin{array}{rl}
\max &J = \int_0^{x_1} y \, dx \\
\text{subject to} & \int_0^{x_1} \sqrt{1 + y'^2} \, dx = c\\
\text{and}& y(0) = y(x_1) = 0
\end{array}
$$
We could convert this to an unconstrained problem using a Lagrange multiplier method;
$$\max K = \int_0^{x_1} L(y,y') \, dx \equiv \int_0^{x_1} y \, dx + u \int_0^{x_1} \sqrt{1+y'^2} \, dx $$
with boundary conditions ##y(0) = y(x_1) = 0##.
Here, the real constant ##u## is the Lagrange multiplier and ##L(y,y') = y + u \sqrt{1 + y'^2}.##

But you can convert [itex]\int y dx = \int y \frac{dx}{ds} ds = \int y \sqrt{1-(\frac{dy}{ds})^2} ds[/itex]. Presumably, that is equivalent to doing the Lagrange multiplier thing.
 
  • #5
stevendaryl said:
But you can convert [itex]\int y dx = \int y \frac{dx}{ds} ds = \int y \sqrt{1-(\frac{dy}{ds})^2} ds[/itex]. Presumably, that is equivalent to doing the Lagrange multiplier thing.

Yes, but that is not the formulation originally given in the question, which is to maximize ##c^{-1} \int_0^{l} y \sqrt{1+y'(s)^2} \, ds##. When I said I did not understand the formulation, that is the one I was referring to.
 
  • #6
Ray Vickson said:
Yes, but that is not the formulation originally given in the question, which is to maximize ##c^{-1} \int_0^{l} y \sqrt{1+y'(s)^2} \, ds##. When I said I did not understand the formulation, that is the one I was referring to.

Oh, I didn't notice the sign. I think the sign of [itex]y'[/itex]. I think the sign is wrong in the original post.
 

Related to Calculus of Variations; Maximum enclosed area problem.

What is the calculus of variations?

The calculus of variations is a branch of mathematics that deals with finding the optimal solution for a given function, equation, or system. It involves the use of differential and integral calculus to find the minimum or maximum value of a functional.

What is the maximum enclosed area problem?

The maximum enclosed area problem is a classic problem in the calculus of variations that involves finding the largest area that can be enclosed by a given length of fence. It is often used as an introductory example to demonstrate the concepts of the calculus of variations.

How is the maximum enclosed area problem solved?

The maximum enclosed area problem can be solved using the Euler-Lagrange equation, which is a necessary condition for finding the optimal solution in the calculus of variations. By applying this equation, the optimal shape of the enclosed area can be determined.

What are the real-world applications of the maximum enclosed area problem?

The maximum enclosed area problem has many real-world applications, including finding the optimal shape of a garden or field with a fixed amount of fencing, minimizing the cost of material for packaging, and optimizing the shape of a wing for maximum lift in aerodynamics.

What are the limitations of the maximum enclosed area problem?

The maximum enclosed area problem assumes that the shape of the enclosed area is continuous and differentiable, which may not always be the case in real-world scenarios. It also does not take into account external factors, such as wind or terrain, which may affect the optimal solution.

Similar threads

  • Calculus and Beyond Homework Help
Replies
13
Views
377
  • Calculus and Beyond Homework Help
Replies
18
Views
1K
  • Calculus and Beyond Homework Help
Replies
5
Views
789
  • Calculus and Beyond Homework Help
Replies
5
Views
668
  • Calculus and Beyond Homework Help
Replies
2
Views
585
  • Calculus and Beyond Homework Help
Replies
2
Views
495
  • Calculus and Beyond Homework Help
Replies
4
Views
737
  • Calculus and Beyond Homework Help
Replies
6
Views
591
Replies
9
Views
759
  • Calculus and Beyond Homework Help
Replies
6
Views
807
Back
Top